Diễn Đàn MathScopeDiễn Đàn MathScope
  Diễn Đàn MathScope
Ghi Danh Hỏi/Ðáp Community Lịch

Go Back   Diễn Đàn MathScope > Sơ Cấp > Đại Số và Lượng Giác > Các Bài Toán Đã Được Giải

News & Announcements

Ngoài một số quy định đã được nêu trong phần Quy định của Ghi Danh , mọi người tranh thủ bỏ ra 5 phút để đọc thêm một số Quy định sau để khỏi bị treo nick ở MathScope nhé !

* Nội quy MathScope.Org

* Một số quy định chung !

* Quy định về việc viết bài trong diễn đàn MathScope

* Nếu bạn muốn gia nhập đội ngũ BQT thì vui lòng tham gia tại đây

* Những câu hỏi thường gặp

* Về việc viết bài trong Box Đại học và Sau đại học


Trả lời Gởi Ðề Tài Mới
 
Ðiều Chỉnh Xếp Bài
Old 21-03-2009, 12:46 PM   #1
-life-
+Thành Viên+
 
Tham gia ngày: Mar 2009
Bài gởi: 2
Thanks: 1
Thanked 0 Times in 0 Posts
Một bài bđt không đẹp - cần giúp

C/m với a,b,c thực dương
$\frac{1}{a(b+1)}+\frac{1}{b(c+1)}+\frac{1}{c(a+1)} \geq \frac{3}{abc+1} $umb:
[RIGHT][I][B]Nguồn: MathScope.ORG[/B][/I][/RIGHT]
 
-life- is offline   Trả Lời Với Trích Dẫn
Old 21-03-2009, 04:36 PM   #2
DCsonlinh_DHV
+Thành Viên+
 
DCsonlinh_DHV's Avatar
 
Tham gia ngày: Mar 2009
Đến từ: *♥*
Bài gởi: 236
Thanks: 32
Thanked 53 Times in 37 Posts
Trích:
Nguyên văn bởi -life- View Post
C/m với a,b,c thực dương
$\frac{1}{a(b+1)}+\frac{1}{b(c+1)}+\frac{1}{c(a+1)} \geq \frac{3}{abc+1} $umb:
bất đẳng thức này dấu bằng xảy ra khi nào nhỉumb: $a=b=c= \pm 1 $ ?
[RIGHT][I][B]Nguồn: MathScope.ORG[/B][/I][/RIGHT]
 
__________________

thay đổi nội dung bởi: DCsonlinh_DHV, 21-03-2009 lúc 04:39 PM
DCsonlinh_DHV is offline   Trả Lời Với Trích Dẫn
Old 21-03-2009, 04:53 PM   #3
Minh Tuấn
+Thành Viên Danh Dự+
 
Tham gia ngày: Jul 2008
Bài gởi: 218
Thanks: 13
Thanked 78 Times in 41 Posts
Thế này mà ko đẹp à
[Only registered and activated users can see links. ]
[RIGHT][I][B]Nguồn: MathScope.ORG[/B][/I][/RIGHT]
 
Minh Tuấn is offline   Trả Lời Với Trích Dẫn
Old 21-03-2009, 04:54 PM   #4
namdung
Administrator

 
Tham gia ngày: Feb 2009
Đến từ: Tp Hồ Chí Minh
Bài gởi: 1,343
Thanks: 209
Thanked 4,066 Times in 778 Posts
Gửi tin nhắn qua Yahoo chát tới namdung
Bài này dùng phân tích sau:

$\frac{abc+1}{a(1+b)} = \frac{1+a}{a(1+b)} + \frac{b(1+c)}{1+b} - 1 $
và các phân tích tương tự, sau đó dùng AM-GM.

Tôi post lại đây trích dẫn lấy từ Mathlinks (hình như từ năm 2005).

This is a famous and well known inequality which appeared in many mathematical journals.The official solution of the proposer was based on calculus. The nicest proof was found by a sixteen student who was praised for it by the proposer. This proof is a "mathematical gem" for its simplicity.

THE PROOF:
Rewrite
(1+abc)/a(1+b)=(1+a)/a(1+b) +b(1+c)/(1+b) -1
(1+abc)/b(1+c)=(1+b)/b(1+c) +c(1+a)/(1+c) -1
(1+abc)/c(1+a)=(1+c)/c(1+a) +a(1+b)/(1+a) -1
Remembering that t + 1/t ≥ 2 we obtain
(1+abc)/a(1+b)+(1+abc)/b(1+c) + (1+abc)/c(1+a) ≥ 2+2+2-3 =3.
[RIGHT][I][B]Nguồn: MathScope.ORG[/B][/I][/RIGHT]
 
namdung is offline   Trả Lời Với Trích Dẫn
The Following User Says Thank You to namdung For This Useful Post:
-life- (21-03-2009)
Old 21-03-2009, 06:30 PM   #5
-life-
+Thành Viên+
 
Tham gia ngày: Mar 2009
Bài gởi: 2
Thanks: 1
Thanked 0 Times in 0 Posts
các bạn phân tích hay thật
mình dùng trung gian toàn ngược dấu mà thấy kì kì về cái chỗ dấu = xảy ra nên chán.
cám ơn nhiều
[RIGHT][I][B]Nguồn: MathScope.ORG[/B][/I][/RIGHT]
 
-life- is offline   Trả Lời Với Trích Dẫn
Old 21-03-2009, 06:44 PM   #6
can_hang2008
+Thành Viên+
 
Tham gia ngày: Mar 2009
Bài gởi: 310
Thanks: 5
Thanked 751 Times in 187 Posts
Trích:
Nguyên văn bởi -life- View Post
C/m với a,b,c thực dương
$\frac{1}{a(b+1)}+\frac{1}{b(c+1)}+\frac{1}{c(a+1)} \geq \frac{3}{abc+1} $umb:
Mình là new member, có gì tệ quá mấy bác bỏ qua cho. Bài này chúng ta có nhiều cách giải, một trong các cách giải mà mình nghĩ là mới là cách sau:
Áp dụng bất đẳng thức $(x+y+z)^2 \ge 3(xy+yz+zx), $ ta thấy rằng chỉ cần chứng minh rằng
$\frac{1}{ab(b+1)(c+1)}+\frac{1}{bc(c+1)(a+1)}+\fra c{1}{ca (a+1)(b+1)} \ge \frac{3}{(abc+1)^2} $
tương đương
$(a+b+c+ab+bc+ca)(abc+1)^2 \ge 3abc(a+1)(b+1)(c+1) $
tức là
$(a+b+c+ab+bc+ca)(a^2b^2c^2-abc+1) \ge 3abc(abc+1) $
Đến đây, áp dụng bất đẳng thức AM-GM, ta thấy $a+b+c+ab+bc+ca \ge 6\sqrt{abc} \ge \frac{12abc}{abc+1} $ nên ta chỉ cần chứng minh được
$\frac{12abc}{abc+1} (a^2b^2c^2-abc+1) \ge 3abc(abc+1) $
hay là
$4(a^2b^2c^2-abc+1) \ge (abc+1)^2 $
Cái này là AM-GM. :hornytoro:
[RIGHT][I][B]Nguồn: MathScope.ORG[/B][/I][/RIGHT]
 

thay đổi nội dung bởi: vipCD, 21-03-2009 lúc 08:14 PM
can_hang2008 is offline   Trả Lời Với Trích Dẫn
Old 21-03-2009, 08:12 PM   #7
vipCD
+Thành Viên Danh Dự+
 
vipCD's Avatar
 
Tham gia ngày: Nov 2007
Bài gởi: 403
Thanks: 34
Thanked 78 Times in 34 Posts
Trích:
Nguyên văn bởi can_hang2008 View Post
Mình là new member, có gì tệ quá mấy bác bỏ qua cho. Bài này chúng ta có nhiều cách giải, một trong các cách giải mà mình nghĩ là mới là cách sau:
Áp dụng bất đẳng thức $(x+y+z)^2 \ge 3(xy+yz+zx), $ ta thấy rằng chỉ cần chứng minh rằng
$\frac{1}{ab(b+1)(c+1)}+\frac{1}{bc(c+1)(a+1)}+\fra c{ 1 } { c a ( a + 1 ) ( b + 1 ) } \ge \frac{3}{(abc+1)^2} $
tương đương
$(a+b+c+ab+bc+ca)(abc+1)^2 \ge 3abc(a+1)(b+1)(c+1) $
tức là
$(a+b+c+ab+bc+ca)(a^2b^2c^2-abc+1) \ge 3abc(abc+1) $
Đến đây, áp dụng bất đẳng thức AM-GM, ta thấy $a+b+c+ab+bc+ca \ge 6\sqrt{abc} \ge \frac{12abc}{abc+1} $ nên ta chỉ cần chứng minh được
$\frac{12abc}{abc+1} (a^2b^2c^2-abc+1) \ge 3abc(abc+1) $
hay là
$4(a^2b^2c^2-abc+1) \ge (abc+1)^2 $
Cái này là AM-GM. :hornytoro:

Chào mừng anh Cẩn đến với MS
Anh giải đúng hic, :pflaster:, hi vọng anh sẽ post nhiều bài hay! Cám ơn anh
Ps: anh gõ sao mà cái chỗ kia sữa mãi chả được!
[RIGHT][I][B]Nguồn: MathScope.ORG[/B][/I][/RIGHT]
 
__________________
TRY

thay đổi nội dung bởi: vipCD, 21-03-2009 lúc 08:15 PM
vipCD is offline   Trả Lời Với Trích Dẫn
Old 23-03-2009, 05:35 AM   #8
dduclam
+Thành Viên Danh Dự+
 
dduclam's Avatar
 
Tham gia ngày: Nov 2007
Đến từ: Đại học Sư phạm Hà Nội
Bài gởi: 481
Thanks: 63
Thanked 168 Times in 92 Posts
Gửi tin nhắn qua Yahoo chát tới dduclam
Sử dụng $(x+y+z)^2 \ge 3(xy+yz+zx) $ ta cũng chứng minh được bất đẳng thức mạnh hơn là:
$\frac1{a(b+1)}+\frac1{b(c+1)}+\frac1{c(a+1)} $$\ge \frac{3}{(1+\sqrt[3]{abc}).\sqrt[3]{abc}} $

Ps. LATEX của diễn đàn đang bị lỗi hay sao ấy.
[RIGHT][I][B]Nguồn: MathScope.ORG[/B][/I][/RIGHT]
 
__________________
Một chút cho tâm hồn bay xa

thay đổi nội dung bởi: dduclam, 23-03-2009 lúc 09:09 AM
dduclam is offline   Trả Lời Với Trích Dẫn
Old 23-03-2009, 06:41 AM   #9
can_hang2008
+Thành Viên+
 
Tham gia ngày: Mar 2009
Bài gởi: 310
Thanks: 5
Thanked 751 Times in 187 Posts
Trích:
Nguyên văn bởi dduclam View Post
Sử dụng $(x+y+z)^2 \ge 3(xy+yz+zx) $ ta cũng chứng minh được bất đẳng thức mạnh hơn là:
$\frac1{a(b+1)}+\frac1{b(c+1)}+\frac1{c(a+1)}\ge\fr ac{3}{(1+\sqrt[3]{abc}).\sqrt[3]{abc}} $
Cái này là do $a+b+c+ab+bc+ca \ge 3\sqrt[3]{abc}+3\sqrt[3]{a^2b^2c^2} $ và hàm $f(x)=\frac{x}{x+a} $ là hàm đồng biến với mọi $a>0 $ :hornytoro:
[RIGHT][I][B]Nguồn: MathScope.ORG[/B][/I][/RIGHT]
 
can_hang2008 is offline   Trả Lời Với Trích Dẫn
Old 23-03-2009, 09:04 AM   #10
dung_toan78
+Thành Viên+
 
Tham gia ngày: Jan 2008
Bài gởi: 111
Thanks: 117
Thanked 41 Times in 25 Posts
Gửi tin nhắn qua Yahoo chát tới dung_toan78
Trích:
Nguyên văn bởi can_hang2008 View Post
Cái này là do $a+b+c+ab+bc+ca \ge 3\sqrt[3]{abc}+3\sqrt[3]{a^2b^2c^2} $ và hàm $f(x)=\frac{x}{x+a} $ là hàm đồng biến với mọi $a>0 $ :hornytoro:
Định mỗi năm thay một nick hả CẨN!:hornytoro:
[RIGHT][I][B]Nguồn: MathScope.ORG[/B][/I][/RIGHT]
 
dung_toan78 is offline   Trả Lời Với Trích Dẫn
Old 23-03-2009, 02:34 PM   #11
can_hang2008
+Thành Viên+
 
Tham gia ngày: Mar 2009
Bài gởi: 310
Thanks: 5
Thanked 751 Times in 187 Posts
Hihi, em cũng định thế anh ạ. reamer: Có điều hơi muộn tí, năm 2009 thì lập 2008, năm 2010 thì lập 2009 :kiss:
[RIGHT][I][B]Nguồn: MathScope.ORG[/B][/I][/RIGHT]
 
can_hang2008 is offline   Trả Lời Với Trích Dẫn
Old 25-03-2009, 06:48 PM   #12
can_hang2008
+Thành Viên+
 
Tham gia ngày: Mar 2009
Bài gởi: 310
Thanks: 5
Thanked 751 Times in 187 Posts
Trích:
Nguyên văn bởi -life- View Post
C/m với a,b,c thực dương
$\frac{1}{a(b+1)}+\frac{1}{b(c+1)}+\frac{1}{c(a+1)} \geq \frac{3}{abc+1} $umb:
Anh Gemath đã có một lời giải bằng hình học rất hay cho bài này. Mọi người xem ở đây nhé: [Only registered and activated users can see links. ]
[RIGHT][I][B]Nguồn: MathScope.ORG[/B][/I][/RIGHT]
 

thay đổi nội dung bởi: can_hang2008, 25-03-2009 lúc 06:52 PM
can_hang2008 is offline   Trả Lời Với Trích Dẫn
Trả lời Gởi Ðề Tài Mới

Bookmarks


Quuyền Hạn Của Bạn
You may not post new threads
You may not post replies
You may not post attachments
You may not edit your posts

BB code is Mở
Smilies đang Mở
[IMG] đang Mở
HTML đang Tắt

Chuyển đến


Múi giờ GMT. Hiện tại là 09:48 PM.


Powered by: vBulletin Copyright ©2000-2024, Jelsoft Enterprises Ltd.
Inactive Reminders By mathscope.org
[page compression: 86.13 k/99.44 k (13.39%)]